Nouvelle démonstration du grand théorème de Fermat - Page 5
Discussion fermée
Page 5 sur 8 PremièrePremière 5 DernièreDernière
Affichage des résultats 121 à 150 sur 237

Nouvelle démonstration du grand théorème de Fermat



  1. #121
    leg

    Re : Nouvelle démonstration du grand théorème de Fermat


    ------

    Citation Envoyé par zinia Voir le message
    Mille excuses !
    On montre de manière identique que c,d, et c²+d² sont premiers entre eux et donc qu'ils sont chacun un carré.
    Il existe donc e,f et g tels que c=e²,d=f²,c²+d²=g²
    Soit encore g²=e^4+f^4

    Enfin si je ne me suis pas encore fourvoyé...
    Mais si: x =c²-d² =(c+d)(c-d) alors x serait encore un carré a²..? puisque c et d sont carré et premiers entre eux, donc leur produit est un carré ou je fais une érreur
    zinia ?

    -----

  2. #122
    invite636fa06b

    Re : Nouvelle démonstration du grand théorème de Fermat

    Citation Envoyé par leg Voir le message
    Mais si: x =c²-d² =(c+d)(c-d) alors x serait encore un carré a²..? puisque c et d sont carré et premiers entre eux, donc leur produit est un carré ou je fais une érreur
    zinia ?
    Non, on s'appuie sur le fait que y² est un produit de nombre premiers entre eux qui sont donc des carrés en utilisant le lemme4 de ton post 102. Mais le fait que x soit la différence de deux puissances quatrième ne permet pas de dire que c'est un carré

  3. #123
    leg

    Re : Nouvelle démonstration du grand théorème de Fermat

    exact,
    donc deux cas se posent puisque c² + d² = g² qui est l'hypoténuse, si x = c² - d² n'est pas un carré lorsque l'ypoténuse en est un alors le cas N=4 est résolu ! puisque dans ce cas g^4 = y^4 + x^4 n'existe pas d'où la seule hypothèse qui reste, c'est de dire que x est carré,
    d'où la contradiction qui en découle
    c²+d²=g²
    c²-d²=x²
    deux triplets Pytha dnné avec les mêmes paramettres mais surtout c est le côté impair avec l'hypoténuse g , puis cet lhypoténuse avec x le côté impair
    un triplet pythagoricien X , Y et Z ne peut avoir X=Z et donner deux solutions par addition et soustraction avec le même paramettre, n et p ;sans que n impair change de valeur.
    non..?

  4. #124
    leg

    Re : Nouvelle démonstration du grand théorème de Fermat

    bonjour Zinia, j'espère que mon raisonnement ne te fait pas gratter la tête..
    c'est en lisant ce lemme4, que j'ai trouvé cette contradiction, qui me parraissait évidente .
    .............................. .............................. ................
    Les lemmes 2 et 4 permet d'affirmer que p, q et p2 - q2 sont des carrés. Posons p = m2 , q = n2 et p2 - q2 = r2. Ainsi :
    r2 = p2 - q2 = m4 - n4 = (m2 + n2)(m2 - n2) ;
    .............................. .............................. ................
    (m2 + n2)(m2 - n2) : un triplet pythagoricien qui donne deux solutions n et pair, m ne peut pas être l'hypoténuse et le côté opposé à l'hypoténuse , donc l'auteur aurait pu déjà s'arréter là et conclure le cas N=4,
    et il me semble que Fermat ne pouvait pas, ne pas trouver cette contradiction.

    Alors Gaetan ou tu en ai
    en relisant tes différentes réponses
    lorsque tu parts du triangle rectangle,tu vérifies l'égalité pour N = 2 mais ensuite tu passe à N=3 ou p4 peu importe c'est toujour avec les mêmes entiers x, y et z que tu élèves à la puissance >, mais comment tu montres qu'avec 3 autres entiers différents qui ne sont pas solution de N=2 et qui pourrait par conséquent être solution dans N=3 que ce n'est pas le cas et ce quelque soit N>.

  5. #125
    Gaétan Mbama

    Re : Nouvelle démonstration du grand théorème de Fermat

    Skydancer, salut

    Nous sommes ici à un forum et le débat est scientifique (n’est ce pas Martini ?). Pendant la discussion, nous pouvons nous permettre certaines plaisanteries mais, de grâce, faisons les dans les limites de la courtoisie.

    Non, les gars, je ne vous prends pas pour des… comme tu l’a dis.

    Non, je veux simplement que tu me dises pourquoi, justement, il absurde de vouloir qu’un triplet d’entiers distincts et non nuls (x,y,z) qui vérifie l’égalité x+y=z puissent aussi vérifier l’équation x²+y²=z² et inversement. (Ecartes-moi ton truc trivial sur le binôme, ça ne t’emmènera pas généraliser. Dis moi comment, avec ça, tu pourras me montrer que si x²+y²=z² alors x^3+y^3=z^3 est impossible).

    L’absurdité dont il est question pour ce qui est des égalités x+y=z et x²+y²=z² est du genre 2xy=0. C’est évident (pardon, trivial : on aime l’adjectif !). Je sais que tu le sais. Mais j’aimerai que tu me dise pourquoi, justement, on arrive à cette absurdité.

    Attention, n’ailles pas me chercher le truc qui est à la mode sur l’anneau des entiers p-adiques et autres qu’un élève de la troisième ni même celui du niveau prépa ne pourra pas piger en 30 minutes. Donnes moi plutôt une explication cohérente et simple qui généralise et qu’un bon lycéen pourra comprendre sans pourtant faire violence à ses neurones.

    Je te dis, déjà, que avec l’hypothèse des triangles je peux tout expliquer en deux mots. Je te dire qu’il y a un lien entre l’exposant n et le triplet (x,y,z), que quand vous changer l’exposant, le triplet aussi automatiquement (sauf bien sûr pour n=2. Tu sais pourquoi ?).

    Rvz, salut !

    J’ai l’impression que la trigo n’est pas trop ton truc. M…ouaih, je comprend, avec ces sinus…
    Mais ça ne doit pas être une raison pour que tu rejettes avec horreur et sans arguments valables mon hypothèse sur les triangles. Argumentes, si tu me convaincs, je vais jeter l’éponge.
    Lors de mon dernier post, je m’étais trompé, (1,1,0) n’est pas une solution triviale.

    Evariste_galois, bonjour !

    Tu ne comprends toujours pas pourquoi je m’attache à cette hypothèse des triangles ?

    Que représente pour toi l’équation x^(-2)+y^(-2)=z^(-2) ?
    Ne dis pas que c’est une choucroute ( rvz, le mot est de toi). Intègres plutôt dans ton raisonnement l’hypothèse des triangles et tu comprendra. (Titre 7.3, pages 15 et 16).

    Questions ?

    Cordialement !

  6. #126
    leg

    Re : Nouvelle démonstration du grand théorème de Fermat

    Citation Envoyé par Gaétan Mbama Voir le message
    Skydancer, salut

    Nous sommes ici à un forum et le débat est scientifique (n’est ce pas Martini ?). Pendant la discussion, nous pouvons nous permettre certaines plaisanteries mais, de grâce, faisons les dans les limites de la courtoisie.

    Non, je veux simplement que tu me dises pourquoi, justement, il est absurde de vouloir qu’un triplet d’entiers distincts et non nuls (x,y,z) qui vérifie l’égalité x+y=z puissent aussi vérifier l’équation x²+y²=z² et inversement. (Ecartes-moi ton truc trivial sur le binôme, ça ne t’emmènera pas généraliser. Dis moi comment, avec ça, tu pourras me montrer que si x²+y²=z² alors x^3+y^3=z^3 est impossible).

    martini et zinia ont indiqué que les trois mêmes entiers ne pouvaient être solution dans 2 puissances différentes

    son truc trivial sur le binôme généralise pour les trois même entiers x, y et z mais pas pour trois autres a,b et c ; ce qui n'apporte pas grand chose vu sous cet angle


    L’absurdité dont il est question pour ce qui est des égalités x+y=z et x²+y²=z² est du genre 2xy=0. C’est évident (pardon, trivial : on aime l’adjectif !). Je sais que tu le sais. Mais j’aimerai que tu me dise pourquoi, justement, on arrive à cette absurdité.

    ça, cela doit être interressant

    Attention, n’ailles pas me chercher le truc qui est à la mode sur l’anneau des entiers p-adiques et autres qu’un élève de la troisième ni même celui du niveau prépa ne pourra pas piger en 30 minutes. Donnes moi plutôt une explication cohérente et simple qui généralise et qu’un bon lycéen pourra comprendre sans pourtant faire violence à ses neurones.

    peut être difficile

    Je te dis, déjà, que avec l’hypothèse des triangles je peux tout expliquer en deux mots. Je peux te dire qu’il y a un lien entre l’exposant n et le triplet (x,y,z), que quand vous changer l’exposant, le triplet aussi automatiquement (sauf bien sûr pour n=2. Tu sais pourquoi ?).

    là il y a justement peut être le problème de restriction que laisse sous entendre Martini, et il t'a reposé des questions...

    est ce que tu sous entend par là, que n'importe quel triplet d'entiers serait représenté et ne pourrait satisfaire l'équation de Fermat quelque soit N > 2,3,4...n?

    Rvz, salut !

    Lors de mon dernier post, je m’étais trompé, (1,1,0) n’est pas une solution triviale.

    Evariste_galois, bonjour !

    Tu ne comprends toujours pas pourquoi je m’attache à cette hypothèse des triangles ?

    Que représente pour toi l’équation x^(-2)+y^(-2)=z^(-2) ?
    Ne dis pas que c’est une choucroute ( rvz, le mot est de toi). Intègres plutôt dans ton raisonnement l’hypothèse des triangles et tu comprendra. (Titre 7.3, pages 15 et 16).

    Questions ?

    Cordialement !
    que donne les réponses aux questions que t'a demandé Martini,
    j'attend la suite merci gaetan A+

  7. #127
    Gaétan Mbama

    Re : Nouvelle démonstration du grand théorème de Fermat

    Soyons sérieux les gars !

    Voilà comment je vois les choses.

    J’affirme que le GTF est un cas particulier qui intègre un autre théorème plus général, plus vaste.

    Voilà comment je l’explique :

    1°)L’égalité (ou équation) de Fermat (x^n+y^n=z^n) n’est pas le GTF ( x^n+y^n=z^n impossible pour tout n>2). Il est important de faire ce distinguo. A moins que l’on me dise le contraire !

    2°) Quand j’examine l’égalité de Fermat, je peux le faire sous plusieurs angles, c’est à dire je suis libre d’adopter toutes les hypothèses que je veux concernant les nombres x, y, z et n.

    3°) Le GTF résulte donc d’une tentative de résoudre l’équation de Fermat sous un angle particulier. Et cet angle particulier ce sont les conditions qu’on inflige au triplet (x,y,z) d’être des nombres entiers distincts et non nuls et à l’exposant n d’être un entier non nul.

    Moi j’arrive au GTF en adoptant l’hypothèse des triangles rectangles.

    Alors pourquoi doit-on vouloir que, la première hypothèse qui considère les entiers x, y et z comme des nombres abstraits soit recevable et, la deuxième qui les considère comme mesurant les côtés d’un triangle rectangle ne le soit pas ?

    Question : Pourquoi m’interdit-on cette hypothèse ?

    J’exige une réponse argumentée.

    On ne fait pas les maths avec des dogmes s.v.p.

    PS : C’est bizarre, les gars, quand je vois un nombre, je l’associe à un segment, et quand j’en vois 3, je pense à trois segments pouvant être reliés, entre autres, en triangle. Il me semble que Poincaré (Henri) a raison quand il écrit qu’on naît géomètre ou analyste, pas les deux à la fois. (Martini, tu veux la référence ?Tiens j’ai visité hier ta biblio en ligne. Je l’ai trouvée intéressante. Dis, est ce que tu peux me trouver un bouquin sur les maths de l’antiquité. Il paraît que Pythagore, Archimède et les autres n’étaient que des plagiaires de la science Egyptienne et que le théorème de Pythagore était connu des prêtres egyptiens des millénaires avant sa naissance et que le nombre pi …)

    Amicalement !

  8. #128
    Gaétan Mbama

    Re : Nouvelle démonstration du grand théorème de Fermat

    Leg, je vais y répondre!
    un peu de patience

  9. #129
    erik

    Re : Nouvelle démonstration du grand théorème de Fermat

    2°) Quand j’examine l’égalité de Fermat, je peux le faire sous plusieurs angles, c’est à dire je suis libre d’adopter toutes les hypothèses que je veux concernant les nombres x, y, z et n.
    La seule hypothese du GTF est que x,y,z et n sont entier (et évidemment non triviaux), Rajouter d'autres hypothèses revient à démontrer autre chose que le GTF

  10. #130
    Gaétan Mbama

    Re : Nouvelle démonstration du grand théorème de Fermat

    Martini,

    La formule (c) est fausse. C'est vrai je ne dis pas le contraire.

    Mais je dis ceci :vouloir la démonter entraîne la formule (b) qui qui est bien la formule logique du GTF (page 3)

    tu es d'accord?

  11. #131
    invite7863222222222
    Invité

    Re : Nouvelle démonstration du grand théorème de Fermat

    Alors pourquoi doit-on vouloir que, la première hypothèse qui considère les entiers x, y et z comme des nombres abstraits soit recevable et, la deuxième qui les considère comme mesurant les côtés d’un triangle rectangle ne le soit pas ?

    Question : Pourquoi m’interdit-on cette hypothèse ?
    Heuu... car le triangle rectangle implique une restriction, un cas particuler et le fait de considérer x, y, z et n quelconque c'est une généralisation.

  12. #132
    Gaétan Mbama

    Re : Nouvelle démonstration du grand théorème de Fermat

    jreeman
    Peux tu me spécifier, en termes claires, quelle est cette restriction?

    Et quand on éxamine l'équation de Fermat suivant l'hypothèse des entiers distincts et non nuls, ne fait-on pas là aussi une restriction? Cette restriction je la trouve d'ailleurs non justifiée.

  13. #133
    invite636fa06b

    Re : Nouvelle démonstration du grand théorème de Fermat

    Bonjour,

    Pour leg
    Ce que j'ai dit c'est qu'un triplet de nombres (réels ou entiers ça n'a pas d'importance) strictement positifs (donc non nuls et non négatifs) ne peuvent vérifier la relation de Fermat pour deux exposants différents (les exposants sont supposés également strictement positifs).
    NB : avec des 0 et des 1, on trouve des triplets qui la vérifient pour tout n
    NB2 : comme l'a dit Martini_bird, c'est une question d'ordre de grandeur.
    Voilà la démo compréhensible par un lycéen :
    On suppose que x,y,z vérifie
    On peut supposer, sans nuire à la généralité que .
    Posons alors On a bien a>1 et
    Les deux équations deviennent alors
    et
    Et en réunissant les deux
    Si t=1 on vérifie que cette égalité est impossible
    Avec t et a supérieurs à 1 cette égalité n'est également pas possible car la fonction f x-->x^a est concave, ce qui se traduit par l'inégalité (c-a)f(b)<(b-a)f(c)+(c-b)f(a) où a,b,c sont trois points distincts
    Appliqué aux points 1,t,1+t, cela donne :
    ou encore
    on montre facilement que la fraction est supérieure à 1 comme a et t le sont...
    Comme les inégalités sont au sens strict, il ne peut donc y avoir la double relation

  14. #134
    erik

    Re : Nouvelle démonstration du grand théorème de Fermat

    jreeman
    Peux tu me spécifier, en termes claires, quelle est cette restriction?
    Je me permet de répondre à sa place.

    Le GTF énonce que quelque soit x, y z entiers, l'équation x^n+y^n=z^n n'est jamais verifiée quelque soit n>2.

    Si tu considères que x y et z mesurent les coté d'un triangle rectangle (c'est à dire que x^2+y^2=z^2) tu élimine de ta demonstration un nombre infini de triplet (x,y,z).

    Evidemment si tu prouves que x^2+y^2=z^2 implique le GTF, tu as fait une partie (mais une partie seulement) du travail. Pour finir de prouver le GTF, il te reste à prouver que x^2+y^2<>z^2 implique également le GTF.
    (je note <> pour "différent de")

  15. #135
    invite7863222222222
    Invité

    Re : Nouvelle démonstration du grand théorème de Fermat

    Et quand on éxamine l'équation de Fermat suivant l'hypothèse des entiers distincts et non nuls, ne fait-on pas là aussi une restriction? Cette restriction je la trouve d'ailleurs non justifiée.
    Si justement je trouve pas ca bien ca n'a pas la même portée qu'un résultat avec x, y, z et n reels. Mais dans le cas général je ne vois pas bien ce qu'on peut dire.

    x^n + y^n = z^n avec x, y, z, n réels n'a pas de solutions non plus ?

    Sinon l'hypothèse d'un triangle rectangle restreint, je trouve, tout autant l'intérêt du problème que de considérer x, y, z entiers.

  16. #136
    leg

    Re : Nouvelle démonstration du grand théorème de Fermat

    Citation Envoyé par zinia Voir le message
    Bonjour,

    Pour leg
    Ce que j'ai dit c'est qu'un triplet de nombres (réels ou entiers ça n'a pas d'importance) strictement positifs (donc non nuls et non négatifs) ne peuvent vérifier la relation de Fermat pour deux exposants différents (les exposants sont supposés également strictement positifs).
    NB : avec des 0 et des 1, on trouve des triplets qui la vérifient pour tout n
    NB2 : comme l'a dit Martini_bird, c'est une question d'ordre de grandeur.
    je pense que j'avais bien compris ceci, mais surtout il me serait impossible de mettre en doute ce que tu écris ou martini.
    en tout cas il me parraissait évident pour moi, à première vue, qu'effectivement un même trilplet d'entiers ne pourait vérifier à la fois l'exposant 3 et 7 par exemple, ça c'est clair .
    peut être que j'ai mal exprimé mon idée dans un des post.

    mais en tout les cas, c'est pour celà que je m'interresse au triplets de racines carrées, d'entiers à la puissance n et aux paramettre p et q de ces triplets de racines et non pas aux entiers x , y et z de l'équation de fermat ce qui ne change pas grand chose .
    donc aux côtés du triangle rectangle et aussi à la propriété du cas N =4 que je t'ai montré.
    A+ et merci pour tes interventions

  17. #137
    leg

    Re : Nouvelle démonstration du grand théorème de Fermat

    Citation Envoyé par erik Voir le message
    Je me permet de répondre à sa place.

    Le GTF énonce que quelque soit x, y z entiers, l'équation x^n+y^n=z^n n'est jamais verifiée quelque soit n>2.

    Si tu considères que x y et z mesurent les coté d'un triangle rectangle (c'est à dire que x^2+y^2=z^2) tu élimine de ta demonstration un nombre infini de triplet (x,y,z).

    Evidemment si tu prouves que x^2+y^2=z^2 implique le GTF, tu as fait une partie (mais une partie seulement)
    bonjour erik
    c'est marrant mais j'ai un doute sur ta réponse , et je pense que gaétan n'a pas tort, en utilisant les triangle rectangle mais sous réserve.

    tu es d'accord que si je prend
    les racines carrées de trois entiers quelconques x ,y et z élevée à la puissance N>2 impair et première,
    j'ai trois nombres a, b et c
    si je montre que a² + b² = c² alors il existe forcément x, y et z qui vérifie xN+YN=zN
    alors on peu en convenir a , b et c sont bien les côté d'un triangle rectangle, de plus ces trois nombre ne sont pas n'importe lesquel, ce sont les racines carrées de l'équation de fermat.
    soit je travail avec x, y et z soit avec les racines carrées a,b et c de x,y et z
    en quoi cela pourrait être restrictif ?

  18. #138
    erik

    Re : Nouvelle démonstration du grand théorème de Fermat

    Dans mon post je ne parle que d'entiers.

  19. #139
    invite636fa06b

    Re : Nouvelle démonstration du grand théorème de Fermat

    Citation Envoyé par leg Voir le message
    bonjour erik
    tu es d'accord que si je prend
    les racines carrées de trois entiers quelconques x ,y et z élevée à la puissance N>2 impair et première,
    j'ai trois nombres a, b et c
    Juste une demande d'explication : tu poses ?

  20. #140
    leg

    Re : Nouvelle démonstration du grand théorème de Fermat

    Citation Envoyé par erik Voir le message
    Dans mon post je ne parle que d'entiers.
    ben oui et alors ,
    ça ne répond pas à la question du triangle rectangle est ce oui ou non restrictif car si c'est le cas gaetan à tort sinon il a raison et sa loi des sinus et cos représenterai bien les racines carrées de x, y et z à la puissance n
    donc du GTF
    mais peut être qu'au lieu de dir ...bla etc il faudrait qu'il dise que celà représent les côté d'un triangle rectangle donc les racines carrées de l'equation de fermat.
    non?

  21. #141
    leg

    Re : Nouvelle démonstration du grand théorème de Fermat

    Citation Envoyé par zinia Voir le message
    Juste une demande d'explication : tu poses ?
    a racine carree de x^n
    par exemplx=2
    2^3 = 8 et a= squart8

  22. #142
    invite636fa06b

    Re : Nouvelle démonstration du grand théorème de Fermat

    D'accord avec toi, leg, si tu acceptes comme cotés les nombres que tu as définis, le fait de raisonner sur un triangle rectangle ne perdra pas de généralité.
    La question qui se pose est de savoir comment les reconnaitre.
    Tes a,b,c ne seront plus des entiers mais des réels algébriques.
    Il te faudrait montrer qu'il n'existe pas de triplet de réels, coté d'un triangle rectangle tels que leurs carrés soit une puissance Nième d'un entier (avec le meme N pour les trois).
    Cela démontrerait le GTF dans toute sa généralité. Mais je n'ai pas l'impression que c'est cela qu'a prouvé Gaétan

  23. #143
    leg

    Re : Nouvelle démonstration du grand théorème de Fermat

    Citation Envoyé par Gaétan Mbama Voir le message
    Soyons sérieux les gars !


    Question : Pourquoi m’interdit-on cette hypothèse ?

    J’exige une réponse argumentée.

    On ne fait pas les maths avec des dogmes s.v.p.

    PS : C’est bizarre, les gars, quand je vois un nombre, je l’associe à un segment, et quand j’en vois 3, je pense à trois segments pouvant être reliés, entre autres, en triangle.
    !
    rassure toi on ne t'interdit pas cette hypothèse,
    c'est qu'elle est pas tout à fait bien expliquée, je parle des côtés du triangle rectangle [C’est bizarre, les gars, quand je vois 3 nombres,]moi je le voit comme trois racines carrées qui mesure un triangle rectangle et par conséquent dans l'équation de Fermat cela réprésente bien la racine carrée de x,y et z à la puissance n>2 mais ces nombres, ces à dires ces racines carrées ne peuvent être entieres puisque l'on ne vérifie que les exposants prmiers >2
    mais si dans ton travail tu as dit que ces côtés du triangle rectangle sont des entiers alors tu ne vérifies que les puissances N pair ce qui ne sert à rien,

    ("pour la bonne et simple raison qu'une racine carrée entière d'un entier à la puissance n quelconque > 2 ne peut se trouver que dans les puissance n pair d'où il existerait trois entiers carrés tel que u², v² et w² qui verifieraient (u²)^n + (v²)^n =(w²)^n, pour n premier>2
    = (x^n)² +(y^n)² =(z^n)² et où les racines carrées a ,b et c sont bien égales à x^n ,y^n et z^n
    et donc ton triangle rectangle de côté a , b et c d'hypoténuse, nest pas rectangle donc les égalités ci dessu sont fausses, mais que pour certains entiers qui serait le produit de puissance N>2...")
    sous réserve que je ne sois pas

  24. #144
    leg

    Re : Nouvelle démonstration du grand théorème de Fermat

    Citation Envoyé par zinia Voir le message
    D'accord avec toi, leg, si tu acceptes comme cotés les nombres que tu as définis, le fait de raisonner sur un triangle rectangle ne perdra pas de généralité.
    La question qui se pose est de savoir comment les reconnaitre.
    Tes a,b,c ne seront plus des entiers mais des réels algébriques.
    Il te faudrait montrer qu'il n'existe pas de triplet de réels, coté d'un triangle rectangle tels que leurs carrés soit une puissance Nième d'un entier (avec le meme N pour les trois).
    Cela démontrerait le GTF dans toute sa généralité. Mais je n'ai pas l'impression que c'est cela qu'a prouvé Gaétan
    a)
    tout à fait d'accord avec toi c'est pour cela que je vient de poster une réponse à gaétan est ce que sa loi des cos et sin représente bien les racines carrées = réel algébrique.
    b)
    pour les reconnâitres "pas de problème"
    soit on est capable de montrer qu'il est impossible de paramettrer celon la formule des triplets pythagoriciens un triplet de racines carrée algébrique et de là que le triangle n'est pas rectangle , montrer une contradiction au paramettrage, en prenant des exemple pour N=2 ,puis N+1 ,N+2 par induction on peu peut être ensuite généraliser si la démo et identique avec la même contradiction pour n ,N+1 et n+2

    un exemple de raisonnement:
    il existe x^3 + y^3 = z^3
    donc il existe 3 racines a,b et c qui sont pythagorique alors il existe deux nombre n et p qui paramettre a, b et c
    il existe donc n'> n et p'>p qui peuvent me redonner trois solution > soi:
    1)
    n'=a et p'=b qui au carré et par addition, me donne c²= z^3

    2)par soustraction
    n'=c et p'=b qui au carré et par soustraction me donne a² = x^3

    3)par soustraction
    n'=c et p' =a qui au carré ,par soutraction me donne b² =y^3
    donc trois possibilté de reconstruire un autre Triplet > de racines carrées algébrique pour N=3
    mais regarde ce que donne ma démo du cas n=4:

    lorsque je met n' et p' au carré ilsont deux cubes qui me donne un cube par addit z^3 1) mais par soustraction qu'est ce que j'obtient si et seulement si j'obtient un cube et bien la même contradiction que n=4;
    deux cubes me donne par addit et soustrac un cube avec un seul parramettrage ce que l'on à déjà démontré impossible dans n=2; pour solution dans n=4 et cela est générale quelque soit n=3, 7 ..etc ou N premier
    alors pour que ma démo soit fauuse, il me faut pas être sur que j'obtienne ce cube.
    mais voilà : deux possibilité s'offre à moi
    n'² est un cube, p'² aussi on sait que d'apres pythagore,
    que (Z + Y)(Z-Y) = X² = (p² - q²)²
    donc dans mon premier cas mes deux réels algébriques vont me donner :
    Z^3= n'² + p'² = a² + b² = c² et
    a² - b² = (a+b) (a-b) = u racine carré supposé être celle d'un cube
    mais n'² -p'² = (c+b) (c-b) =c² - b² = x^3 entier or c² et lypothénuse Z et b² et Y ,est ce que je ne devrait pas avoir ce cube aussi carré? en gardant la propriété de pythagore des l'instant ou n' et p' sont pythagorique c'est a dire qu'ils ont été choisis dans un triplet pyhtagoricien de racine carrée ???
    de plus si le produit de deux nombre me donne un cube c'est deux nombres ne sont ils pas des cubes, ou le fait qu'il soit deux racines algébriques est suffisant?
    car si ilsont des cubes, il y a contradiction et surtout le raisonnement qui en découle et génerale :

    je ne pourrai pas choisir n et p dans les racine carrées pour reconstruire une solution qui est supposée exister, donc il n'existe pas de triplet de racines carrées d'une equation de Fermat!
    voila ou j'en suis Zinia

  25. #145
    leg

    Re : Nouvelle démonstration du grand théorème de Fermat

    suite , zinia la seule contradiction que j'ai à ce jour, mais que je suppose insuffisante c'est :

    pour les puissance pair par ex 6

    mes racines "algébrique" là, elles son entièrres et ma démo de N 4, fonctionnne; mais le fait que pour reconstruire la solution supposée exister dans n = 6 et et qui me donne une solution dans n =2 avec aussi une solution dans n=3
    qui sont trois carrés A, b et C à la puissance 3.
    je choisis n et p donc comme pour N =2 dans les racines carrées mais de N=3 car il est supposé exister des solution et là lorsque je met n et p au carré pour obtenir un triplet de racines carrée je me retrouve avec deux solutions systématiquement, par addit et soustracti ce qui est absurde ou alors il est impossible que j'en trouve, donc je ne peux pas reconstruire une solution supposée exister dans N = 6 , 3 et 2 donc je ne peux pas choisir dans les racines carrées de N > 2 donc dans les entiers à la puissance première y compris N = 2 est ce suffisant pour dire alors, qu'il en est de même pour les racines algébriques c'était peut être l'idée merveilleuse de Fermat...? car il existerait ,au moin une solution dans N = 4

    et deuxième contradiction si n et p sont algébrique ou autres irrationels pars ex, mais non entiers il ne peuvent donner une solution dans N = 2 donc quelque soit N pair
    car sinon les trois démos de N= 4 sont fausses ce qui est impossible.il existerait une solution dans N=4 ou du moins, deux carrés possibles dans un triplet pythagoricien

    plus tard, j'irai lui demader je

  26. #146
    leg

    Re : Nouvelle démonstration du grand théorème de Fermat

    ils ne peuvent donner de solutions primitives dans N=2 bien entendu
    et la contradiction pour N4,puis N6 et générale pour N pair.

  27. #147
    leg

    Re : Nouvelle démonstration du grand théorème de Fermat

    la nuit à été courte
    Bonjour à tous
    Salut zinia, peut être que gaétan en utilisant les triangles rectangles retombe sur une propriété de surface. Lorsque l’on regarde les surfaces carrées d’un T.P (triplet pytha), par ex 3.4 et 5, les trois carrés exécuté sur les côtés du triangle rectangle, on obtient 9 carrés de 1 + 16carrés de 1 = 25 carrés de 1 ou 2 T.P de 3.4 et5 mais pour les 2 cas le facteur et le même.
    Il est donc évident que je ne peux choisir qu’un des 3 facteurs du triplet mais en aucun cas, un facteur différent pour chaque côté du triangle rectangle et en espérant obtenir une égalité de surface.
    Fermat à du s’inspirer de cette idée, déjà pour résoudre de façon générale le cas des puissances pairs. Car en effet le Triplet de racines carrées entières se trouve par obligation dans les puissances N premières en commençant par N=2 .
    On constate facilement que tous les T.P sont représenté dans ces entiers élevés à la puissance N.
    Je reprend 3.4 et 5 je ne pourrait le dupliquer que par 3,4 ou 5 mais pas chaque côté par un facteur différent si je duplique par 3 j’obtient bien pour 1 côté X=3² , mais pour Y = 12 et non 16 et pou Z 15 et non 25 que je duplique par 4 ou 5 c’est évident que cela ne peux rien me donner.
    Si je passe à N =3 il en est de même, faire un triangle rectangle avec les 3 côtés qui sont des carrés, est absurde et comme facteur 3, 4 et 5. Soit X = 3 3² ou X = 3 et le facteur 3² ou encore X = 3² et le facteur 3 mais pour Y = 4 le facteur ne peut être que 3 ou 3² et pas 4² etc .si le Triplet exister dans N=2 ou 3, alors il existerait des triplets pythagoriciens pouvant avoir les surfaces carrées dupliquée par des facteurs différents, dont la somme des surfaces des deux côtés du triangle rectangle et = à celles des surfaces de l’hypoténuse ce qui ne voudrait plus rien dire.
    De ce point de vue là il est clair qu’il ne peut y avoir de solution dans N pair = 2 puisque toutes solutions dans N pair > 2 et aussi solution dans cette dernière.

    Ce qui nous montre une autre remarque un T.P de racines carrées entières, ne peut donc pas être solution dans cette même puissance. C’est à dire si il y a une solution dans N= 2 alors il ne peut y avoir de Triplet de racine carrée de N=2
    Si il y a un Triplet de racine entières, donc un Triangle rectangle dans N=3; alors il ne peut y avoir de solution dans N = 3, ce qui serait impossible puisqu’une solution de N=6 , l’ai aussi dans N=2 et 3 .
    De façon générale, il ne peut y avoir de solution dans N > 2 et pair !
    Donc si la propriété développé par Gaetan avec les triangles rectangles, ne faisait apparaître comme côté que des produits de puissance N premiers puisque N pair n’apporte rien , il ne pourrait que dans un premier temps résoudre le cas générale que je viens de citer .

    Enfin et dernière question, si un Triplet de racines entières n’existe pas, cela entraîne t’il les racines carrées donc algébriques de ces même racines carrées entière
    J’entend par là, racine carrée de la racine carrée de l’entier à la puissance N pair ;ex:

    √√26 = √23

  28. #148
    leg

    Re : Nouvelle démonstration du grand théorème de Fermat

    Salut Gaetan.
    2eme explication
    Je suppose, que comme tu l’as dit, tu part du triangle rectangle où les côtés sont bien les racines carrées soit entières ou algébriques, restons sur les entières.
    Tu arrives à démontrer que lorsque les côtés du triangle rectangle deviennent des entiers à la puissance N>1 je dit bien 1, alors ton triangle n’est plus rectangle donc tu ne solutionne que l’égalité du cas N=2 et l’inégalité pour N>2 et pair, à juste raison puisque tu peux représenté les côté du triangle rectangle avec des racine carrée entières donc des produits de puissance N >1 et premiers ; comme tu l’as indiqué et tu as bien représenté l’égalité de Fermat pour cette puissance 2, et l’inégalité pour 4.6. ..n pair.

    Donc tu t’aperçois maintenant, que tu as affaire à des triangles ou scalènes, 3 cotés quelconques.
    Il te faut maintenant montrer pour N premier >2 .
    Deux possibilités :
    a)
    Soit ta démonstration qui est juste dans le fond (« mais fausse, à priori dans le raisonnement faute de complément d’explication») doit montrer qu’il n’est pas possible de tracer un triangle rectangle avec des réel algébrique = racine carrée d’entiers non nul, produit de puissance N premières tel que c’est défini, (« voir les posts plus haut, de Zinia et moi même »).

    b)
    il est impossible que les côtés d’un scalène mesuré par des entiers X, Y et Z non nul, qui sont les produits d’une puissance N 1 et première, ce qui est suffisant ; soit solution de Fermat.
    (« Et non pas de simple entiers non nul a, b et c qui ne vérifie pas X + Y = Z donc, tel que: aN = X ; bN = Y et cN = Z ce que tu dois indiqué, en début de ta démo ainsi que pour les réel algébriques définis»)
    Autrement dit, les côtés X,Y et Z d’un scalène ne peuvent pas être solution de Fermat tel que X + Y ≠ Z
    Ce qui t'oblige obligatoirement tel que c'est défini , de démarrer tes scalènes dans la puissance N=3 , 5,,,N premier
    Est ce bien cela que tu as démontré ?
    [(« et si oui, alors c’est toi qui m’invite au Resto avec Zinia aussi, car elle est de bon conseil ») ]
    A+

  29. #149
    invite79d10163

    Re : Nouvelle démonstration du grand théorème de Fermat

    Bon je vais essayer de reprendre le fil de la discussion...

  30. #150
    indian58

    Re : Nouvelle démonstration du grand théorème de Fermat

    Bonjour à tous,

    je m'excuse de me lancer dans cette discussion aussi tard et je m'excuse donc de ne pas avoir lu tous les posts. J'ai lu les 4 premières pages de posts et la 8 et 9° (mais ai survolé les autres), ainsi que la démonstration de Gaetan. J'ai une question à lui poser, cette question a déjà été posée mais a été, je crois, sans réponse. Cette question me semble donc très importante et je propose qu'on (Gaetan et quiconque y pourrait) y réponde sans plus tarder:

    UNE BONNE FOIS POUR TOUTES, GAETAN, DIS-NOUS OU TU UTILISES LE FAIT QUE X,Y,Z SONT DES ENTIERS.

    Hormis cette question, j'ai quelques remarques à faire sur cette démonstration : à vouloir trop bien expliquer, tu rends le texte confus d'autant plus que tu oublies (ou sous-entends) assez souvent des mots voire des phrases qui auraient pu rendre ce texte plus cohérent (Par exemple, tu oublies de temps en temps le mot "rectangle" ce qui rend la proposition dont tu parles fausse dans le cas général, bien que vraie dans ce cas particulier). Sans compter les quelques erreurs que tu fais (les fonctions que tu étudies sont convexes pas concaves). Je suis prêt à croire que tu as trouvé une démonstration élémentaire du GTF mais explique les points importants plutôt que des points sans importance (plusieurs pages auraient pu être résumées en une seule avec le binôme de Newton). Je te recommande de réécrire ta démonstration.

    En tout cas, si quelqu'un pouvait répondre à cette question, moi ainsi que toutes les autres personnes intéressées lui en serons reconnaissant.

Page 5 sur 8 PremièrePremière 5 DernièreDernière

Discussions similaires

  1. Une démonstration analytique du théorème de Fermat
    Par invitede6b99d6 dans le forum Mathématiques du supérieur
    Réponses: 16
    Dernier message: 29/09/2011, 13h15
  2. Généralisation du grand théorème de Fermat
    Par breukin dans le forum Mathématiques du supérieur
    Réponses: 15
    Dernier message: 02/09/2006, 19h31
  3. Réponses: 7
    Dernier message: 21/06/2005, 02h26
  4. Le dernier théorème de Fermat.
    Par Antikhippe dans le forum Lectures scientifiques
    Réponses: 41
    Dernier message: 02/09/2004, 18h43